Quantcast
  • Register
PhysicsOverflow is a next-generation academic platform for physicists and astronomers, including a community peer review system and a postgraduate-level discussion forum analogous to MathOverflow.

Welcome to PhysicsOverflow! PhysicsOverflow is an open platform for community peer review and graduate-level Physics discussion.

Please help promote PhysicsOverflow ads elsewhere if you like it.

News

PO is now at the Physics Department of Bielefeld University!

New printer friendly PO pages!

Migration to Bielefeld University was successful!

Please vote for this year's PhysicsOverflow ads!

Please do help out in categorising submissions. Submit a paper to PhysicsOverflow!

... see more

Tools for paper authors

Submit paper
Claim Paper Authorship

Tools for SE users

Search User
Reclaim SE Account
Request Account Merger
Nativise imported posts
Claim post (deleted users)
Import SE post

Users whose questions have been imported from Physics Stack Exchange, Theoretical Physics Stack Exchange, or any other Stack Exchange site are kindly requested to reclaim their account and not to register as a new user.

Public \(\beta\) tools

Report a bug with a feature
Request a new functionality
404 page design
Send feedback

Attributions

(propose a free ad)

Site Statistics

205 submissions , 163 unreviewed
5,047 questions , 2,200 unanswered
5,345 answers , 22,709 comments
1,470 users with positive rep
816 active unimported users
More ...

  How should we deal with diagrams which do not conserve particle number in a non-relativistic field theory?

+ 3 like - 0 dislike
917 views

In the last 10 years there has been more and more crossover of techniques from high energy physics being used in AMO and condensed matter scenarios, in particular diagrammatic techniques and related perturbative calculations. Unlike the relativistic case, a simple Schrodinger field theory will conserve the number of particles; however, not all diagrams that one might draw respect this conserved quantity. My question is, what should be done with such diagrams? Will the theory generically result in these diagrams not contributing to scattering processes or is this something that has to be input 'by hand?'

As an illustrative example, consider the interaction of two non-relativistic scalar fields with a local quartic interaction 
$$\mathcal{L} = \sum_{i=1}^2{(\psi^\dagger_i \dot{\psi}_i - \frac{1}{2m} \nabla \psi^\dagger_i \nabla \psi_i)} + \frac{g}{m} \psi^\dagger_1 \psi^\dagger_2 \psi_2 \psi_1,$$ which represents a system whose excitations are fermions with 2 orthogonal spin states with a contact interaction. Such a theory is considered in Braaten and Platter's [Exact Relations for a Strongly Interacting Fermi Gas from the Operator Product Expansion](http://journals.aps.org/prl/pdf/10.1103/PhysRevLett.100.205301).

Note in particular that they find that after imposing a momentum cutoff for the theory, the coupling constant must satisfy $$g(\Lambda) = \frac{4 \pi a}{1- 2 a \Lambda/\pi}$$ to recover the scattering amplitude we expect from the zero-range model with a scattering length $a$ (and $\Lambda$ is the momentum cutoff). Importantly, the coupling constant $g \sim \mathcal{O}(1/\Lambda)$ for large cutoff.

However, if one computes the corrections to the propagator (of either particle) you find that the first order correction has only one diagram and its value is $$\Delta_1(k) \sim -\frac{i g}{8 m \pi^2} \frac{\Lambda^3}{3}.$$ On the contrary, I expected this diagram to not contribute anything (similarly to how it does not in the relativistic $\phi^4$ theory)  1) because it does not conserve particle number and 2) because the parameter 'm' of the bare theory is the SAME 'm' that appears in the equivalent Schrodinger equation, meaning that the physical mass of the particle is not changed by the perturbative corrections. And then, not only is the contribution non-zero but it blows up as the cutoff becomes large. Are we required to postulate that $$m = m_0 + \mathcal{O}(\Lambda^3)$$ so that these contributions do go to zero as the cutoff increases?

Identical question for reference: http://physics.stackexchange.com/questions/200237/how-should-we-deal-with-diagrams-which-do-not-conserve-particle-number-in-a-non

asked Aug 20, 2015 in Theoretical Physics by Kevin Driscoll [ no revision ]

If this diagram is a correction to the propagator how does it not conserve particle number? The initial and final particle number should still both be 1, right? In relativistic \(\phi^4\) theory there is most definitely a self-energy contribution that blows up with the cutoff.

1 Answer

+ 0 like - 0 dislike

I can't imagine which self-energy diagram you are considering. An important occurring in the non-relativistic theory (and in the absence of chemical potentials) is that there is no correction to the propagator to any order in perturbation theory. 

answered Oct 28, 2015 by anonymous [ no revision ]

Your answer

Please use answers only to (at least partly) answer questions. To comment, discuss, or ask for clarification, leave a comment instead.
To mask links under text, please type your text, highlight it, and click the "link" button. You can then enter your link URL.
Please consult the FAQ for as to how to format your post.
This is the answer box; if you want to write a comment instead, please use the 'add comment' button.
Live preview (may slow down editor)   Preview
Your name to display (optional):
Privacy: Your email address will only be used for sending these notifications.
Anti-spam verification:
If you are a human please identify the position of the character covered by the symbol $\varnothing$ in the following word:
p$\hbar$y$\varnothing$icsOverflow
Then drag the red bullet below over the corresponding character of our banner. When you drop it there, the bullet changes to green (on slow internet connections after a few seconds).
Please complete the anti-spam verification




user contributions licensed under cc by-sa 3.0 with attribution required

Your rights
...